对齐:将左侧对齐到左侧,并在中心运算符上居中对齐

对齐:将左侧对齐到左侧,并在中心运算符上居中对齐

目前,我有类似这样的事情:

\begin{align*}
    \varsigma, \iota, t \models_{\fim} \texttt{p} \  \texttt{s}_{\texttt{i}_0} \ldots \texttt{s}_{\texttt{i}_n} 
    & : \Leftrightarrow \ 
    \eta_{\fim}\varsigma, \iota, t, \texttt{p} \ \texttt{s}_{\texttt{i}_0} \ldots \texttt{s}_{\texttt{i}_n}  \\
    %
    \varsigma, \iota, t \models_{\fim} [\texttt{s} \triangleleft \sigi] 
    &: \Leftrightarrow \ 
    \varsigma(t)(\texttt{s}) \equiv \sigi  \\
    %
    \varsigma, \iota, t \models_{\fim} \neg \varphi 
    &: \Leftrightarrow \ 
    \varsigma, \iota, t \not \models_{\fim} \neg \varphi  \\
    %
    \varsigma, \iota, t \models_{\fim} \varphi \wedge \psi
    &: \Leftrightarrow \ 
    \varsigma, \iota, t \models_{\fim} \varphi \wedge \varsigma, \iota, t \models_{\fim} \psi 
\end{align*}

其结果是:

在此处输入图片描述

但我想要的是类似这样的东西:

在此处输入图片描述

如果有重复,很高兴被重定向,但如果我没记错的话,我还没有看到要求左对齐和居中对齐的帖子。

注意:flalign似乎没有做任何事情。

答案1

使用alignat

在此处输入图片描述

\documentclass{article}

\usepackage{amsmath}

\newcommand{\fim}{\langle \cdot \rangle}

\begin{document}

\begin{alignat*}{2}
  \varsigma, \iota, t & \models_{\fim} \texttt{p} \ \texttt{s}_{\texttt{i}_0} \ldots \texttt{s}_{\texttt{i}_n} 
  && : \Leftrightarrow \ 
  \eta_{\fim}\varsigma, \iota, t, \texttt{p} \ \texttt{s}_{\texttt{i}_0} \ldots \texttt{s}_{\texttt{i}_n}  \\
  %
  \varsigma, \iota, t & \models_{\fim} [\texttt{s} \triangleleft \texttt{s}_{\texttt{i}}] 
  &&: \Leftrightarrow \ 
  \varsigma(t)(\texttt{s}) \equiv \texttt{s}_{\texttt{i}}  \\
  %
  \varsigma, \iota, t & \models_{\fim} \neg \varphi 
  &&: \Leftrightarrow \ 
  \varsigma, \iota, t \not \models_{\fim} \neg \varphi  \\
  %
  \varsigma, \iota, t & \models_{\fim} \varphi \wedge \psi
  &&: \Leftrightarrow \ 
  \varsigma, \iota, t \models_{\fim} \varphi \wedge \varsigma, \iota, t \models_{\fim} \psi 
\end{alignat*}

\end{document}

相关内容